Finding the nth term of sequence of 3, 10, 31, 94, 283…Finding the nth term in a repeating number sequenceNth term of a sequencefinding nth term in a geometric sequenceFind nth term of sequenceFinding the nth term of a geometric sequenceProve that sequence $S_N(a, n, d) = ±1 pmod N$ if $N$ is prime.Finding the nth term of a numeric sequence- Newton's little formula explanationProof for nth term of the sequenceFinding the $nth$ term of a sequencenth term of the sequence 1,2,3,5,7,9…

Why are all my yellow 2V/20mA LEDs burning out with 330k Ohm resistor?

How many hours would it take to watch all of Doctor Who?

What was the definition of "set" that resulted in Russell's Paradox

What is the job of the acoustic cavities inside the main combustion chamber?

Why are Hobbits so fond of mushrooms?

Confirming the Identity of a (Friendly) Reviewer After the Reviews

Combining latex input and sed

How to wire 3 hots / 3 neutrals into this new TR outlet with 2 screws?

Would dual wielding daggers be a viable choice for a covert bodyguard?

Is "I do not want you to go nowhere" a case of "DOUBLE-NEGATIVES" as claimed by Grammarly?

What's the point of having a RAID 1 configuration over incremental backups to a secondary drive?

Print the last, middle and first character of your code

Does throwing a penny at a train stop the train?

How to memorize multiple pieces?

Why isn't pressure filtration popular compared to vacuum filtration?

Can the Mage Hand cantrip be used to trip an enemy who is running away?

If a non-friend comes across my Steam Wishlist, how easily can he gift me one of the games?

C program to parse source code of another language

Referring to different instances of the same character in time travel

How to tell someone I'd like to become friends without letting them think I'm romantically interested in them?

What does 「はった」 mean?

How can one write good dialogue in a story without sounding wooden?

Should disabled buttons give feedback when clicked?

How to loop for 3 times in bash script when docker push fails?



Finding the nth term of sequence of 3, 10, 31, 94, 283…


Finding the nth term in a repeating number sequenceNth term of a sequencefinding nth term in a geometric sequenceFind nth term of sequenceFinding the nth term of a geometric sequenceProve that sequence $S_N(a, n, d) = ±1 pmod N$ if $N$ is prime.Finding the nth term of a numeric sequence- Newton's little formula explanationProof for nth term of the sequenceFinding the $nth$ term of a sequencenth term of the sequence 1,2,3,5,7,9…






.everyoneloves__top-leaderboard:empty,.everyoneloves__mid-leaderboard:empty,.everyoneloves__bot-mid-leaderboard:empty margin-bottom:0;








1












$begingroup$


How do you work out the nth term of a sequence that's neither arithmetic or geometric where two operations are used to get from one term to the next? For example, the sequence 3, 10, 31, 94, 283 where each term is the last term multiply 3 plus 1. (i.e $u_1 = 3$ and $u_n+1 = 3u_n + 1$). Then is there a way to generalise it where $u_1 = a$ and $u_n+1 = bu_n + c$?



This problem was part of a question on the Oxford MAT. I've tried to find common differences, substitute terms and many other methods. Also I found that this problem fits in an area of maths called recurrence relations but the Wikipedia page was far too confusing for me since I'm only year 11 (grade 10), so it didn't help answer my question. I wonder if there's a simpler explanation for this problem. Thanks in advance










share|cite|improve this question









New contributor



FailToWinPRO is a new contributor to this site. Take care in asking for clarification, commenting, and answering.
Check out our Code of Conduct.






$endgroup$







  • 2




    $begingroup$
    The differences are $$7,21=3cdot 7,63=9cdot 7,189=27cdot 7$$
    $endgroup$
    – Dr. Sonnhard Graubner
    9 hours ago


















1












$begingroup$


How do you work out the nth term of a sequence that's neither arithmetic or geometric where two operations are used to get from one term to the next? For example, the sequence 3, 10, 31, 94, 283 where each term is the last term multiply 3 plus 1. (i.e $u_1 = 3$ and $u_n+1 = 3u_n + 1$). Then is there a way to generalise it where $u_1 = a$ and $u_n+1 = bu_n + c$?



This problem was part of a question on the Oxford MAT. I've tried to find common differences, substitute terms and many other methods. Also I found that this problem fits in an area of maths called recurrence relations but the Wikipedia page was far too confusing for me since I'm only year 11 (grade 10), so it didn't help answer my question. I wonder if there's a simpler explanation for this problem. Thanks in advance










share|cite|improve this question









New contributor



FailToWinPRO is a new contributor to this site. Take care in asking for clarification, commenting, and answering.
Check out our Code of Conduct.






$endgroup$







  • 2




    $begingroup$
    The differences are $$7,21=3cdot 7,63=9cdot 7,189=27cdot 7$$
    $endgroup$
    – Dr. Sonnhard Graubner
    9 hours ago














1












1








1





$begingroup$


How do you work out the nth term of a sequence that's neither arithmetic or geometric where two operations are used to get from one term to the next? For example, the sequence 3, 10, 31, 94, 283 where each term is the last term multiply 3 plus 1. (i.e $u_1 = 3$ and $u_n+1 = 3u_n + 1$). Then is there a way to generalise it where $u_1 = a$ and $u_n+1 = bu_n + c$?



This problem was part of a question on the Oxford MAT. I've tried to find common differences, substitute terms and many other methods. Also I found that this problem fits in an area of maths called recurrence relations but the Wikipedia page was far too confusing for me since I'm only year 11 (grade 10), so it didn't help answer my question. I wonder if there's a simpler explanation for this problem. Thanks in advance










share|cite|improve this question









New contributor



FailToWinPRO is a new contributor to this site. Take care in asking for clarification, commenting, and answering.
Check out our Code of Conduct.






$endgroup$




How do you work out the nth term of a sequence that's neither arithmetic or geometric where two operations are used to get from one term to the next? For example, the sequence 3, 10, 31, 94, 283 where each term is the last term multiply 3 plus 1. (i.e $u_1 = 3$ and $u_n+1 = 3u_n + 1$). Then is there a way to generalise it where $u_1 = a$ and $u_n+1 = bu_n + c$?



This problem was part of a question on the Oxford MAT. I've tried to find common differences, substitute terms and many other methods. Also I found that this problem fits in an area of maths called recurrence relations but the Wikipedia page was far too confusing for me since I'm only year 11 (grade 10), so it didn't help answer my question. I wonder if there's a simpler explanation for this problem. Thanks in advance







sequences-and-series






share|cite|improve this question









New contributor



FailToWinPRO is a new contributor to this site. Take care in asking for clarification, commenting, and answering.
Check out our Code of Conduct.










share|cite|improve this question









New contributor



FailToWinPRO is a new contributor to this site. Take care in asking for clarification, commenting, and answering.
Check out our Code of Conduct.








share|cite|improve this question




share|cite|improve this question








edited 9 hours ago









Greg Martin

37.9k2 gold badges35 silver badges66 bronze badges




37.9k2 gold badges35 silver badges66 bronze badges






New contributor



FailToWinPRO is a new contributor to this site. Take care in asking for clarification, commenting, and answering.
Check out our Code of Conduct.








asked 9 hours ago









FailToWinPROFailToWinPRO

82 bronze badges




82 bronze badges




New contributor



FailToWinPRO is a new contributor to this site. Take care in asking for clarification, commenting, and answering.
Check out our Code of Conduct.




New contributor




FailToWinPRO is a new contributor to this site. Take care in asking for clarification, commenting, and answering.
Check out our Code of Conduct.









  • 2




    $begingroup$
    The differences are $$7,21=3cdot 7,63=9cdot 7,189=27cdot 7$$
    $endgroup$
    – Dr. Sonnhard Graubner
    9 hours ago













  • 2




    $begingroup$
    The differences are $$7,21=3cdot 7,63=9cdot 7,189=27cdot 7$$
    $endgroup$
    – Dr. Sonnhard Graubner
    9 hours ago








2




2




$begingroup$
The differences are $$7,21=3cdot 7,63=9cdot 7,189=27cdot 7$$
$endgroup$
– Dr. Sonnhard Graubner
9 hours ago





$begingroup$
The differences are $$7,21=3cdot 7,63=9cdot 7,189=27cdot 7$$
$endgroup$
– Dr. Sonnhard Graubner
9 hours ago











5 Answers
5






active

oldest

votes


















6












$begingroup$

There's a nice trick for recursive sequences of this type, where $u_n+1$ is a linear function of $u_n$: find a constant $r$ such that $u_n+1-r$ is a constant multiple of $u_n-r$. In this case, the multiplication factor in the linear function is $3$, so we're searching for an $r$ with $u_n+1-r = 3(u_n-r)$. Solving for $r$:
beginalign*
u_n+1-r &= 3(u_n-r) \
(3u_n+1)-r &= 3u_n-3r \
1+2r &= 0 \
r &= -tfrac12.
endalign*



Why does this help us? Because $u_n+1+frac12 = 3(u_n+frac12)$, it's easy to prove by induction that $u_n+frac12 = 3^n-1(u_1+frac12)$ for all $nge1$. (In other words, this slightly shifted version of the sequence really is geometric.) Solving for $u_n$:
beginalign*
u_n+tfrac12 &= 3^n-1(u_1+tfrac12) \
u_n &= 3^n-1(3+tfrac12)-tfrac12 \
u_n &= tfrac12(7cdot3^n-1-1).
endalign*

(Of course there's some formula we could memorize that gives the answer immediately, which is fine ... but it's always best to know how such formulas are derived in the first place.)






share|cite|improve this answer









$endgroup$












  • $begingroup$
    Thanks, that result seems correct. However, although it's probably obvious, I can't quite wrap my head around the proof by induction. Could you give a detailed step by step of why since $u_n+1 + frac12 = 3(u_n + frac12), u_n + frac12 = 3^n-1(u_1+ frac12)$ is true?
    $endgroup$
    – FailToWinPRO
    8 hours ago



















2












$begingroup$

Observe that
beginalign*
u_n+1&=3u_n+1\
u_n&=3u_n-1+1
endalign*



Then



$$(u_n+1-u_n)=3(u_n-u_n-1).$$



Let us define $a_n=u_n-u_n-1$, then the above equation can be written as
$$a_n+1=3a_n.$$
This gives us



beginalign*
a_3&=3a_2\
vdots & =vdots\
a_n+1&=3a_n
endalign*

If we multiply these out, we get



$$a_n+1=3^n-1a_2$$



So we have
$$u_n+1-u_n=3^n-1(u_2-u_1)=3^n-1 ,7.$$



Now add these:



beginalign*
u_2-u_1 &=3^0 ,7\
u_3-u_2 &=3^1 ,7\
u_4-u_3 &=3^2 ,7\
vdots & =vdots\
u_n+1-u_n &=3^n-1 ,7
endalign*

To get
$$u_n+1-u_1=7(3^0+3^1+dotsb +3^n-1)$$
Thus,




$$u_n+1=7left(frac3^n-12right)+3 =frac12left(7. , 3^n-1right)quad text for n geq
colorred0.$$

In fact, you can test that validity of this expression by plugging $n=0$ to get $u_1=3$, with $n=1$ we get $u_2=10$ and so on.







share|cite|improve this answer











$endgroup$








  • 1




    $begingroup$
    This doesn't require any tool beyond what you may have already seen in high school mathematics.
    $endgroup$
    – Anurag A
    8 hours ago


















0












$begingroup$

Try writing down the first few terms then observe how the coefficients are truncated. if the first term is $u_1$,



I found that the $n^th$ term is:



$$u_n = b^n-1u_1 + c*sum_i=2^n a^n-i$$






share|cite|improve this answer









$endgroup$












  • $begingroup$
    In your case, take $u_1=3$, $b = 3$, $c = 1$.
    $endgroup$
    – Book Book Book
    9 hours ago


















0












$begingroup$

Set $v_n=u_n+a$ to try to get the recurrence to be $v_n+1=3v_n$. So, you want
$$u_n+1+a=3u_n+3a$$
that is
$$3u_n+1+a=3u_n+3a.$$
Therefore you want $a=1/2$. So the sequence $v_n$ is $7/2$, $21/2$, $63/2$ etc. Then
$$v_n=3^n-1frac72$$
and
$$u_n=3^n-1frac72-frac12.$$






share|cite|improve this answer









$endgroup$




















    0












    $begingroup$

    I have worked out this sum for your sequence: $$u_n=left (sum_i=0^n3^i right ) - 3^n-1$$. You can easily find it if you substitute recursively values in the formula. For example for $n=4$ it's $3^4+3^2+3^1+1$. I obtain a geometric progression (but there isn't $3^3$) and the sum in general is: $$u_n=frac1-3^n+11-3-3^n-1$$.
    In the end: $$u_n=frac12(3^n+1-1)-3^n-1$$



    For the second question: $$u_n=ab^n-1+sum_i=0^n-1cb^i=ab^n-1+frac(cb)^n-1-1cb-1$$






    share|cite|improve this answer










    New contributor



    Matteo is a new contributor to this site. Take care in asking for clarification, commenting, and answering.
    Check out our Code of Conduct.





    $endgroup$















      Your Answer








      StackExchange.ready(function()
      var channelOptions =
      tags: "".split(" "),
      id: "69"
      ;
      initTagRenderer("".split(" "), "".split(" "), channelOptions);

      StackExchange.using("externalEditor", function()
      // Have to fire editor after snippets, if snippets enabled
      if (StackExchange.settings.snippets.snippetsEnabled)
      StackExchange.using("snippets", function()
      createEditor();
      );

      else
      createEditor();

      );

      function createEditor()
      StackExchange.prepareEditor(
      heartbeatType: 'answer',
      autoActivateHeartbeat: false,
      convertImagesToLinks: true,
      noModals: true,
      showLowRepImageUploadWarning: true,
      reputationToPostImages: 10,
      bindNavPrevention: true,
      postfix: "",
      imageUploader:
      brandingHtml: "Powered by u003ca class="icon-imgur-white" href="https://imgur.com/"u003eu003c/au003e",
      contentPolicyHtml: "User contributions licensed under u003ca href="https://creativecommons.org/licenses/by-sa/3.0/"u003ecc by-sa 3.0 with attribution requiredu003c/au003e u003ca href="https://stackoverflow.com/legal/content-policy"u003e(content policy)u003c/au003e",
      allowUrls: true
      ,
      noCode: true, onDemand: true,
      discardSelector: ".discard-answer"
      ,immediatelyShowMarkdownHelp:true
      );



      );






      FailToWinPRO is a new contributor. Be nice, and check out our Code of Conduct.









      draft saved

      draft discarded


















      StackExchange.ready(
      function ()
      StackExchange.openid.initPostLogin('.new-post-login', 'https%3a%2f%2fmath.stackexchange.com%2fquestions%2f3289168%2ffinding-the-nth-term-of-sequence-of-3-10-31-94-283%23new-answer', 'question_page');

      );

      Post as a guest















      Required, but never shown

























      5 Answers
      5






      active

      oldest

      votes








      5 Answers
      5






      active

      oldest

      votes









      active

      oldest

      votes






      active

      oldest

      votes









      6












      $begingroup$

      There's a nice trick for recursive sequences of this type, where $u_n+1$ is a linear function of $u_n$: find a constant $r$ such that $u_n+1-r$ is a constant multiple of $u_n-r$. In this case, the multiplication factor in the linear function is $3$, so we're searching for an $r$ with $u_n+1-r = 3(u_n-r)$. Solving for $r$:
      beginalign*
      u_n+1-r &= 3(u_n-r) \
      (3u_n+1)-r &= 3u_n-3r \
      1+2r &= 0 \
      r &= -tfrac12.
      endalign*



      Why does this help us? Because $u_n+1+frac12 = 3(u_n+frac12)$, it's easy to prove by induction that $u_n+frac12 = 3^n-1(u_1+frac12)$ for all $nge1$. (In other words, this slightly shifted version of the sequence really is geometric.) Solving for $u_n$:
      beginalign*
      u_n+tfrac12 &= 3^n-1(u_1+tfrac12) \
      u_n &= 3^n-1(3+tfrac12)-tfrac12 \
      u_n &= tfrac12(7cdot3^n-1-1).
      endalign*

      (Of course there's some formula we could memorize that gives the answer immediately, which is fine ... but it's always best to know how such formulas are derived in the first place.)






      share|cite|improve this answer









      $endgroup$












      • $begingroup$
        Thanks, that result seems correct. However, although it's probably obvious, I can't quite wrap my head around the proof by induction. Could you give a detailed step by step of why since $u_n+1 + frac12 = 3(u_n + frac12), u_n + frac12 = 3^n-1(u_1+ frac12)$ is true?
        $endgroup$
        – FailToWinPRO
        8 hours ago
















      6












      $begingroup$

      There's a nice trick for recursive sequences of this type, where $u_n+1$ is a linear function of $u_n$: find a constant $r$ such that $u_n+1-r$ is a constant multiple of $u_n-r$. In this case, the multiplication factor in the linear function is $3$, so we're searching for an $r$ with $u_n+1-r = 3(u_n-r)$. Solving for $r$:
      beginalign*
      u_n+1-r &= 3(u_n-r) \
      (3u_n+1)-r &= 3u_n-3r \
      1+2r &= 0 \
      r &= -tfrac12.
      endalign*



      Why does this help us? Because $u_n+1+frac12 = 3(u_n+frac12)$, it's easy to prove by induction that $u_n+frac12 = 3^n-1(u_1+frac12)$ for all $nge1$. (In other words, this slightly shifted version of the sequence really is geometric.) Solving for $u_n$:
      beginalign*
      u_n+tfrac12 &= 3^n-1(u_1+tfrac12) \
      u_n &= 3^n-1(3+tfrac12)-tfrac12 \
      u_n &= tfrac12(7cdot3^n-1-1).
      endalign*

      (Of course there's some formula we could memorize that gives the answer immediately, which is fine ... but it's always best to know how such formulas are derived in the first place.)






      share|cite|improve this answer









      $endgroup$












      • $begingroup$
        Thanks, that result seems correct. However, although it's probably obvious, I can't quite wrap my head around the proof by induction. Could you give a detailed step by step of why since $u_n+1 + frac12 = 3(u_n + frac12), u_n + frac12 = 3^n-1(u_1+ frac12)$ is true?
        $endgroup$
        – FailToWinPRO
        8 hours ago














      6












      6








      6





      $begingroup$

      There's a nice trick for recursive sequences of this type, where $u_n+1$ is a linear function of $u_n$: find a constant $r$ such that $u_n+1-r$ is a constant multiple of $u_n-r$. In this case, the multiplication factor in the linear function is $3$, so we're searching for an $r$ with $u_n+1-r = 3(u_n-r)$. Solving for $r$:
      beginalign*
      u_n+1-r &= 3(u_n-r) \
      (3u_n+1)-r &= 3u_n-3r \
      1+2r &= 0 \
      r &= -tfrac12.
      endalign*



      Why does this help us? Because $u_n+1+frac12 = 3(u_n+frac12)$, it's easy to prove by induction that $u_n+frac12 = 3^n-1(u_1+frac12)$ for all $nge1$. (In other words, this slightly shifted version of the sequence really is geometric.) Solving for $u_n$:
      beginalign*
      u_n+tfrac12 &= 3^n-1(u_1+tfrac12) \
      u_n &= 3^n-1(3+tfrac12)-tfrac12 \
      u_n &= tfrac12(7cdot3^n-1-1).
      endalign*

      (Of course there's some formula we could memorize that gives the answer immediately, which is fine ... but it's always best to know how such formulas are derived in the first place.)






      share|cite|improve this answer









      $endgroup$



      There's a nice trick for recursive sequences of this type, where $u_n+1$ is a linear function of $u_n$: find a constant $r$ such that $u_n+1-r$ is a constant multiple of $u_n-r$. In this case, the multiplication factor in the linear function is $3$, so we're searching for an $r$ with $u_n+1-r = 3(u_n-r)$. Solving for $r$:
      beginalign*
      u_n+1-r &= 3(u_n-r) \
      (3u_n+1)-r &= 3u_n-3r \
      1+2r &= 0 \
      r &= -tfrac12.
      endalign*



      Why does this help us? Because $u_n+1+frac12 = 3(u_n+frac12)$, it's easy to prove by induction that $u_n+frac12 = 3^n-1(u_1+frac12)$ for all $nge1$. (In other words, this slightly shifted version of the sequence really is geometric.) Solving for $u_n$:
      beginalign*
      u_n+tfrac12 &= 3^n-1(u_1+tfrac12) \
      u_n &= 3^n-1(3+tfrac12)-tfrac12 \
      u_n &= tfrac12(7cdot3^n-1-1).
      endalign*

      (Of course there's some formula we could memorize that gives the answer immediately, which is fine ... but it's always best to know how such formulas are derived in the first place.)







      share|cite|improve this answer












      share|cite|improve this answer



      share|cite|improve this answer










      answered 9 hours ago









      Greg MartinGreg Martin

      37.9k2 gold badges35 silver badges66 bronze badges




      37.9k2 gold badges35 silver badges66 bronze badges











      • $begingroup$
        Thanks, that result seems correct. However, although it's probably obvious, I can't quite wrap my head around the proof by induction. Could you give a detailed step by step of why since $u_n+1 + frac12 = 3(u_n + frac12), u_n + frac12 = 3^n-1(u_1+ frac12)$ is true?
        $endgroup$
        – FailToWinPRO
        8 hours ago

















      • $begingroup$
        Thanks, that result seems correct. However, although it's probably obvious, I can't quite wrap my head around the proof by induction. Could you give a detailed step by step of why since $u_n+1 + frac12 = 3(u_n + frac12), u_n + frac12 = 3^n-1(u_1+ frac12)$ is true?
        $endgroup$
        – FailToWinPRO
        8 hours ago
















      $begingroup$
      Thanks, that result seems correct. However, although it's probably obvious, I can't quite wrap my head around the proof by induction. Could you give a detailed step by step of why since $u_n+1 + frac12 = 3(u_n + frac12), u_n + frac12 = 3^n-1(u_1+ frac12)$ is true?
      $endgroup$
      – FailToWinPRO
      8 hours ago





      $begingroup$
      Thanks, that result seems correct. However, although it's probably obvious, I can't quite wrap my head around the proof by induction. Could you give a detailed step by step of why since $u_n+1 + frac12 = 3(u_n + frac12), u_n + frac12 = 3^n-1(u_1+ frac12)$ is true?
      $endgroup$
      – FailToWinPRO
      8 hours ago














      2












      $begingroup$

      Observe that
      beginalign*
      u_n+1&=3u_n+1\
      u_n&=3u_n-1+1
      endalign*



      Then



      $$(u_n+1-u_n)=3(u_n-u_n-1).$$



      Let us define $a_n=u_n-u_n-1$, then the above equation can be written as
      $$a_n+1=3a_n.$$
      This gives us



      beginalign*
      a_3&=3a_2\
      vdots & =vdots\
      a_n+1&=3a_n
      endalign*

      If we multiply these out, we get



      $$a_n+1=3^n-1a_2$$



      So we have
      $$u_n+1-u_n=3^n-1(u_2-u_1)=3^n-1 ,7.$$



      Now add these:



      beginalign*
      u_2-u_1 &=3^0 ,7\
      u_3-u_2 &=3^1 ,7\
      u_4-u_3 &=3^2 ,7\
      vdots & =vdots\
      u_n+1-u_n &=3^n-1 ,7
      endalign*

      To get
      $$u_n+1-u_1=7(3^0+3^1+dotsb +3^n-1)$$
      Thus,




      $$u_n+1=7left(frac3^n-12right)+3 =frac12left(7. , 3^n-1right)quad text for n geq
      colorred0.$$

      In fact, you can test that validity of this expression by plugging $n=0$ to get $u_1=3$, with $n=1$ we get $u_2=10$ and so on.







      share|cite|improve this answer











      $endgroup$








      • 1




        $begingroup$
        This doesn't require any tool beyond what you may have already seen in high school mathematics.
        $endgroup$
        – Anurag A
        8 hours ago















      2












      $begingroup$

      Observe that
      beginalign*
      u_n+1&=3u_n+1\
      u_n&=3u_n-1+1
      endalign*



      Then



      $$(u_n+1-u_n)=3(u_n-u_n-1).$$



      Let us define $a_n=u_n-u_n-1$, then the above equation can be written as
      $$a_n+1=3a_n.$$
      This gives us



      beginalign*
      a_3&=3a_2\
      vdots & =vdots\
      a_n+1&=3a_n
      endalign*

      If we multiply these out, we get



      $$a_n+1=3^n-1a_2$$



      So we have
      $$u_n+1-u_n=3^n-1(u_2-u_1)=3^n-1 ,7.$$



      Now add these:



      beginalign*
      u_2-u_1 &=3^0 ,7\
      u_3-u_2 &=3^1 ,7\
      u_4-u_3 &=3^2 ,7\
      vdots & =vdots\
      u_n+1-u_n &=3^n-1 ,7
      endalign*

      To get
      $$u_n+1-u_1=7(3^0+3^1+dotsb +3^n-1)$$
      Thus,




      $$u_n+1=7left(frac3^n-12right)+3 =frac12left(7. , 3^n-1right)quad text for n geq
      colorred0.$$

      In fact, you can test that validity of this expression by plugging $n=0$ to get $u_1=3$, with $n=1$ we get $u_2=10$ and so on.







      share|cite|improve this answer











      $endgroup$








      • 1




        $begingroup$
        This doesn't require any tool beyond what you may have already seen in high school mathematics.
        $endgroup$
        – Anurag A
        8 hours ago













      2












      2








      2





      $begingroup$

      Observe that
      beginalign*
      u_n+1&=3u_n+1\
      u_n&=3u_n-1+1
      endalign*



      Then



      $$(u_n+1-u_n)=3(u_n-u_n-1).$$



      Let us define $a_n=u_n-u_n-1$, then the above equation can be written as
      $$a_n+1=3a_n.$$
      This gives us



      beginalign*
      a_3&=3a_2\
      vdots & =vdots\
      a_n+1&=3a_n
      endalign*

      If we multiply these out, we get



      $$a_n+1=3^n-1a_2$$



      So we have
      $$u_n+1-u_n=3^n-1(u_2-u_1)=3^n-1 ,7.$$



      Now add these:



      beginalign*
      u_2-u_1 &=3^0 ,7\
      u_3-u_2 &=3^1 ,7\
      u_4-u_3 &=3^2 ,7\
      vdots & =vdots\
      u_n+1-u_n &=3^n-1 ,7
      endalign*

      To get
      $$u_n+1-u_1=7(3^0+3^1+dotsb +3^n-1)$$
      Thus,




      $$u_n+1=7left(frac3^n-12right)+3 =frac12left(7. , 3^n-1right)quad text for n geq
      colorred0.$$

      In fact, you can test that validity of this expression by plugging $n=0$ to get $u_1=3$, with $n=1$ we get $u_2=10$ and so on.







      share|cite|improve this answer











      $endgroup$



      Observe that
      beginalign*
      u_n+1&=3u_n+1\
      u_n&=3u_n-1+1
      endalign*



      Then



      $$(u_n+1-u_n)=3(u_n-u_n-1).$$



      Let us define $a_n=u_n-u_n-1$, then the above equation can be written as
      $$a_n+1=3a_n.$$
      This gives us



      beginalign*
      a_3&=3a_2\
      vdots & =vdots\
      a_n+1&=3a_n
      endalign*

      If we multiply these out, we get



      $$a_n+1=3^n-1a_2$$



      So we have
      $$u_n+1-u_n=3^n-1(u_2-u_1)=3^n-1 ,7.$$



      Now add these:



      beginalign*
      u_2-u_1 &=3^0 ,7\
      u_3-u_2 &=3^1 ,7\
      u_4-u_3 &=3^2 ,7\
      vdots & =vdots\
      u_n+1-u_n &=3^n-1 ,7
      endalign*

      To get
      $$u_n+1-u_1=7(3^0+3^1+dotsb +3^n-1)$$
      Thus,




      $$u_n+1=7left(frac3^n-12right)+3 =frac12left(7. , 3^n-1right)quad text for n geq
      colorred0.$$

      In fact, you can test that validity of this expression by plugging $n=0$ to get $u_1=3$, with $n=1$ we get $u_2=10$ and so on.








      share|cite|improve this answer














      share|cite|improve this answer



      share|cite|improve this answer








      edited 8 hours ago

























      answered 9 hours ago









      Anurag AAnurag A

      28.1k1 gold badge23 silver badges52 bronze badges




      28.1k1 gold badge23 silver badges52 bronze badges







      • 1




        $begingroup$
        This doesn't require any tool beyond what you may have already seen in high school mathematics.
        $endgroup$
        – Anurag A
        8 hours ago












      • 1




        $begingroup$
        This doesn't require any tool beyond what you may have already seen in high school mathematics.
        $endgroup$
        – Anurag A
        8 hours ago







      1




      1




      $begingroup$
      This doesn't require any tool beyond what you may have already seen in high school mathematics.
      $endgroup$
      – Anurag A
      8 hours ago




      $begingroup$
      This doesn't require any tool beyond what you may have already seen in high school mathematics.
      $endgroup$
      – Anurag A
      8 hours ago











      0












      $begingroup$

      Try writing down the first few terms then observe how the coefficients are truncated. if the first term is $u_1$,



      I found that the $n^th$ term is:



      $$u_n = b^n-1u_1 + c*sum_i=2^n a^n-i$$






      share|cite|improve this answer









      $endgroup$












      • $begingroup$
        In your case, take $u_1=3$, $b = 3$, $c = 1$.
        $endgroup$
        – Book Book Book
        9 hours ago















      0












      $begingroup$

      Try writing down the first few terms then observe how the coefficients are truncated. if the first term is $u_1$,



      I found that the $n^th$ term is:



      $$u_n = b^n-1u_1 + c*sum_i=2^n a^n-i$$






      share|cite|improve this answer









      $endgroup$












      • $begingroup$
        In your case, take $u_1=3$, $b = 3$, $c = 1$.
        $endgroup$
        – Book Book Book
        9 hours ago













      0












      0








      0





      $begingroup$

      Try writing down the first few terms then observe how the coefficients are truncated. if the first term is $u_1$,



      I found that the $n^th$ term is:



      $$u_n = b^n-1u_1 + c*sum_i=2^n a^n-i$$






      share|cite|improve this answer









      $endgroup$



      Try writing down the first few terms then observe how the coefficients are truncated. if the first term is $u_1$,



      I found that the $n^th$ term is:



      $$u_n = b^n-1u_1 + c*sum_i=2^n a^n-i$$







      share|cite|improve this answer












      share|cite|improve this answer



      share|cite|improve this answer










      answered 9 hours ago









      Book Book BookBook Book Book

      4897 bronze badges




      4897 bronze badges











      • $begingroup$
        In your case, take $u_1=3$, $b = 3$, $c = 1$.
        $endgroup$
        – Book Book Book
        9 hours ago
















      • $begingroup$
        In your case, take $u_1=3$, $b = 3$, $c = 1$.
        $endgroup$
        – Book Book Book
        9 hours ago















      $begingroup$
      In your case, take $u_1=3$, $b = 3$, $c = 1$.
      $endgroup$
      – Book Book Book
      9 hours ago




      $begingroup$
      In your case, take $u_1=3$, $b = 3$, $c = 1$.
      $endgroup$
      – Book Book Book
      9 hours ago











      0












      $begingroup$

      Set $v_n=u_n+a$ to try to get the recurrence to be $v_n+1=3v_n$. So, you want
      $$u_n+1+a=3u_n+3a$$
      that is
      $$3u_n+1+a=3u_n+3a.$$
      Therefore you want $a=1/2$. So the sequence $v_n$ is $7/2$, $21/2$, $63/2$ etc. Then
      $$v_n=3^n-1frac72$$
      and
      $$u_n=3^n-1frac72-frac12.$$






      share|cite|improve this answer









      $endgroup$

















        0












        $begingroup$

        Set $v_n=u_n+a$ to try to get the recurrence to be $v_n+1=3v_n$. So, you want
        $$u_n+1+a=3u_n+3a$$
        that is
        $$3u_n+1+a=3u_n+3a.$$
        Therefore you want $a=1/2$. So the sequence $v_n$ is $7/2$, $21/2$, $63/2$ etc. Then
        $$v_n=3^n-1frac72$$
        and
        $$u_n=3^n-1frac72-frac12.$$






        share|cite|improve this answer









        $endgroup$















          0












          0








          0





          $begingroup$

          Set $v_n=u_n+a$ to try to get the recurrence to be $v_n+1=3v_n$. So, you want
          $$u_n+1+a=3u_n+3a$$
          that is
          $$3u_n+1+a=3u_n+3a.$$
          Therefore you want $a=1/2$. So the sequence $v_n$ is $7/2$, $21/2$, $63/2$ etc. Then
          $$v_n=3^n-1frac72$$
          and
          $$u_n=3^n-1frac72-frac12.$$






          share|cite|improve this answer









          $endgroup$



          Set $v_n=u_n+a$ to try to get the recurrence to be $v_n+1=3v_n$. So, you want
          $$u_n+1+a=3u_n+3a$$
          that is
          $$3u_n+1+a=3u_n+3a.$$
          Therefore you want $a=1/2$. So the sequence $v_n$ is $7/2$, $21/2$, $63/2$ etc. Then
          $$v_n=3^n-1frac72$$
          and
          $$u_n=3^n-1frac72-frac12.$$







          share|cite|improve this answer












          share|cite|improve this answer



          share|cite|improve this answer










          answered 9 hours ago









          Lord Shark the UnknownLord Shark the Unknown

          116k11 gold badges67 silver badges147 bronze badges




          116k11 gold badges67 silver badges147 bronze badges





















              0












              $begingroup$

              I have worked out this sum for your sequence: $$u_n=left (sum_i=0^n3^i right ) - 3^n-1$$. You can easily find it if you substitute recursively values in the formula. For example for $n=4$ it's $3^4+3^2+3^1+1$. I obtain a geometric progression (but there isn't $3^3$) and the sum in general is: $$u_n=frac1-3^n+11-3-3^n-1$$.
              In the end: $$u_n=frac12(3^n+1-1)-3^n-1$$



              For the second question: $$u_n=ab^n-1+sum_i=0^n-1cb^i=ab^n-1+frac(cb)^n-1-1cb-1$$






              share|cite|improve this answer










              New contributor



              Matteo is a new contributor to this site. Take care in asking for clarification, commenting, and answering.
              Check out our Code of Conduct.





              $endgroup$

















                0












                $begingroup$

                I have worked out this sum for your sequence: $$u_n=left (sum_i=0^n3^i right ) - 3^n-1$$. You can easily find it if you substitute recursively values in the formula. For example for $n=4$ it's $3^4+3^2+3^1+1$. I obtain a geometric progression (but there isn't $3^3$) and the sum in general is: $$u_n=frac1-3^n+11-3-3^n-1$$.
                In the end: $$u_n=frac12(3^n+1-1)-3^n-1$$



                For the second question: $$u_n=ab^n-1+sum_i=0^n-1cb^i=ab^n-1+frac(cb)^n-1-1cb-1$$






                share|cite|improve this answer










                New contributor



                Matteo is a new contributor to this site. Take care in asking for clarification, commenting, and answering.
                Check out our Code of Conduct.





                $endgroup$















                  0












                  0








                  0





                  $begingroup$

                  I have worked out this sum for your sequence: $$u_n=left (sum_i=0^n3^i right ) - 3^n-1$$. You can easily find it if you substitute recursively values in the formula. For example for $n=4$ it's $3^4+3^2+3^1+1$. I obtain a geometric progression (but there isn't $3^3$) and the sum in general is: $$u_n=frac1-3^n+11-3-3^n-1$$.
                  In the end: $$u_n=frac12(3^n+1-1)-3^n-1$$



                  For the second question: $$u_n=ab^n-1+sum_i=0^n-1cb^i=ab^n-1+frac(cb)^n-1-1cb-1$$






                  share|cite|improve this answer










                  New contributor



                  Matteo is a new contributor to this site. Take care in asking for clarification, commenting, and answering.
                  Check out our Code of Conduct.





                  $endgroup$



                  I have worked out this sum for your sequence: $$u_n=left (sum_i=0^n3^i right ) - 3^n-1$$. You can easily find it if you substitute recursively values in the formula. For example for $n=4$ it's $3^4+3^2+3^1+1$. I obtain a geometric progression (but there isn't $3^3$) and the sum in general is: $$u_n=frac1-3^n+11-3-3^n-1$$.
                  In the end: $$u_n=frac12(3^n+1-1)-3^n-1$$



                  For the second question: $$u_n=ab^n-1+sum_i=0^n-1cb^i=ab^n-1+frac(cb)^n-1-1cb-1$$







                  share|cite|improve this answer










                  New contributor



                  Matteo is a new contributor to this site. Take care in asking for clarification, commenting, and answering.
                  Check out our Code of Conduct.








                  share|cite|improve this answer



                  share|cite|improve this answer








                  edited 7 hours ago





















                  New contributor



                  Matteo is a new contributor to this site. Take care in asking for clarification, commenting, and answering.
                  Check out our Code of Conduct.








                  answered 8 hours ago









                  MatteoMatteo

                  144 bronze badges




                  144 bronze badges




                  New contributor



                  Matteo is a new contributor to this site. Take care in asking for clarification, commenting, and answering.
                  Check out our Code of Conduct.




                  New contributor




                  Matteo is a new contributor to this site. Take care in asking for clarification, commenting, and answering.
                  Check out our Code of Conduct.






















                      FailToWinPRO is a new contributor. Be nice, and check out our Code of Conduct.









                      draft saved

                      draft discarded


















                      FailToWinPRO is a new contributor. Be nice, and check out our Code of Conduct.












                      FailToWinPRO is a new contributor. Be nice, and check out our Code of Conduct.











                      FailToWinPRO is a new contributor. Be nice, and check out our Code of Conduct.














                      Thanks for contributing an answer to Mathematics Stack Exchange!


                      • Please be sure to answer the question. Provide details and share your research!

                      But avoid


                      • Asking for help, clarification, or responding to other answers.

                      • Making statements based on opinion; back them up with references or personal experience.

                      Use MathJax to format equations. MathJax reference.


                      To learn more, see our tips on writing great answers.




                      draft saved


                      draft discarded














                      StackExchange.ready(
                      function ()
                      StackExchange.openid.initPostLogin('.new-post-login', 'https%3a%2f%2fmath.stackexchange.com%2fquestions%2f3289168%2ffinding-the-nth-term-of-sequence-of-3-10-31-94-283%23new-answer', 'question_page');

                      );

                      Post as a guest















                      Required, but never shown





















































                      Required, but never shown














                      Required, but never shown












                      Required, but never shown







                      Required, but never shown

































                      Required, but never shown














                      Required, but never shown












                      Required, but never shown







                      Required, but never shown







                      Popular posts from this blog

                      Invision Community Contents History See also References External links Navigation menuProprietaryinvisioncommunity.comIPS Community ForumsIPS Community Forumsthis blog entry"License Changes, IP.Board 3.4, and the Future""Interview -- Matt Mecham of Ibforums""CEO Invision Power Board, Matt Mecham Is a Liar, Thief!"IPB License Explanation 1.3, 1.3.1, 2.0, and 2.1ArchivedSecurity Fixes, Updates And Enhancements For IPB 1.3.1Archived"New Demo Accounts - Invision Power Services"the original"New Default Skin"the original"Invision Power Board 3.0.0 and Applications Released"the original"Archived copy"the original"Perpetual licenses being done away with""Release Notes - Invision Power Services""Introducing: IPS Community Suite 4!"Invision Community Release Notes

                      Canceling a color specificationRandomly assigning color to Graphics3D objects?Default color for Filling in Mathematica 9Coloring specific elements of sets with a prime modified order in an array plotHow to pick a color differing significantly from the colors already in a given color list?Detection of the text colorColor numbers based on their valueCan color schemes for use with ColorData include opacity specification?My dynamic color schemes

                      Tom Holland Mục lục Đầu đời và giáo dục | Sự nghiệp | Cuộc sống cá nhân | Phim tham gia | Giải thưởng và đề cử | Chú thích | Liên kết ngoài | Trình đơn chuyển hướngProfile“Person Details for Thomas Stanley Holland, "England and Wales Birth Registration Index, 1837-2008" — FamilySearch.org”"Meet Tom Holland... the 16-year-old star of The Impossible""Schoolboy actor Tom Holland finds himself in Oscar contention for role in tsunami drama"“Naomi Watts on the Prince William and Harry's reaction to her film about the late Princess Diana”lưu trữ"Holland and Pflueger Are West End's Two New 'Billy Elliots'""I'm so envious of my son, the movie star! British writer Dominic Holland's spent 20 years trying to crack Hollywood - but he's been beaten to it by a very unlikely rival"“Richard and Margaret Povey of Jersey, Channel Islands, UK: Information about Thomas Stanley Holland”"Tom Holland to play Billy Elliot""New Billy Elliot leaving the garage"Billy Elliot the Musical - Tom Holland - Billy"A Tale of four Billys: Tom Holland""The Feel Good Factor""Thames Christian College schoolboys join Myleene Klass for The Feelgood Factor""Government launches £600,000 arts bursaries pilot""BILLY's Chapman, Holland, Gardner & Jackson-Keen Visit Prime Minister""Elton John 'blown away' by Billy Elliot fifth birthday" (video with John's interview and fragments of Holland's performance)"First News interviews Arrietty's Tom Holland"“33rd Critics' Circle Film Awards winners”“National Board of Review Current Awards”Bản gốc"Ron Howard Whaling Tale 'In The Heart Of The Sea' Casts Tom Holland"“'Spider-Man' Finds Tom Holland to Star as New Web-Slinger”lưu trữ“Captain America: Civil War (2016)”“Film Review: ‘Captain America: Civil War’”lưu trữ“‘Captain America: Civil War’ review: Choose your own avenger”lưu trữ“The Lost City of Z reviews”“Sony Pictures and Marvel Studios Find Their 'Spider-Man' Star and Director”“‘Mary Magdalene’, ‘Current War’ & ‘Wind River’ Get 2017 Release Dates From Weinstein”“Lionsgate Unleashing Daisy Ridley & Tom Holland Starrer ‘Chaos Walking’ In Cannes”“PTA's 'Master' Leads Chicago Film Critics Nominations, UPDATED: Houston and Indiana Critics Nominations”“Nominaciones Goya 2013 Telecinco Cinema – ENG”“Jameson Empire Film Awards: Martin Freeman wins best actor for performance in The Hobbit”“34th Annual Young Artist Awards”Bản gốc“Teen Choice Awards 2016—Captain America: Civil War Leads Second Wave of Nominations”“BAFTA Film Award Nominations: ‘La La Land’ Leads Race”“Saturn Awards Nominations 2017: 'Rogue One,' 'Walking Dead' Lead”Tom HollandTom HollandTom HollandTom Hollandmedia.gettyimages.comWorldCat Identities300279794no20130442900000 0004 0355 42791085670554170004732cb16706349t(data)XX5557367